Integral of a triangular pyramid

  • Thread starter Thread starter doctordiddy
  • Start date Start date
  • Tags Tags
    Integral Pyramid
Click For Summary
To find the volume of a pyramid with a height of 24 and a base of an equilateral triangle with a side of 11, the user attempts to set up an integral for the area A(x) of the triangular cross-section. They correctly identify the relationship between the dimensions but struggle with the area formula, mistakenly including too many factors of l in their calculations. Simplifying the expression for the area of the triangle will help clarify the integral setup. Once corrected, the integral should become more manageable to solve. The discussion emphasizes the need for accurate area calculations in integral applications for volume.
doctordiddy
Messages
53
Reaction score
0

Homework Statement



Find the volume of a pyramid with height 24 and with base an equilateral triangle with side 11.

Homework Equations

The Attempt at a Solution



So I know the relationship

h/x=b/l

where h is my height 24
x is simply x or height above base
b is my base 11
l is the length of the base at height x

i also know the integral is

024(A(x))dx

I am having trouble finding A(x)

I believe that it you would use the area of the equilateral triangle

so i know area of the triangle is base*height/2

I also know that base is l, and height is unknown, but i do know that

l^2=(l/2)^2 + z^2 where i take z as the height of the equilateral triangle

then z= √(l^2 -(l/2)^2 )

so area of the triangle would finall be

[(l^2)√(l^2 - (l/2)^2)]/2

I would finally plug in l=bx/h from the identity h/x=b/l

However, when i do this, i end up with an integral i am unable to solve. I feel like there is another way to do this, can anyone help?

edit: I mean an easier way using integrals, not volume of a pyramid please

thanks
 
Physics news on Phys.org
What kind of a triangle is an equilateral triangle?
 
doctordiddy said:
[(l^2)√(l^2 - (l/2)^2)]/2
You've one too many l factors in there - it's an area, not a volume. You should be able to simplify the term inside the square root (bringing the l outside). Once you've done that the integral should be straightforward.
 
Question: A clock's minute hand has length 4 and its hour hand has length 3. What is the distance between the tips at the moment when it is increasing most rapidly?(Putnam Exam Question) Answer: Making assumption that both the hands moves at constant angular velocities, the answer is ## \sqrt{7} .## But don't you think this assumption is somewhat doubtful and wrong?

Similar threads

  • · Replies 7 ·
Replies
7
Views
1K
Replies
4
Views
2K
  • · Replies 30 ·
2
Replies
30
Views
4K
  • · Replies 9 ·
Replies
9
Views
2K
  • · Replies 10 ·
Replies
10
Views
2K
  • · Replies 2 ·
Replies
2
Views
2K
  • · Replies 20 ·
Replies
20
Views
2K
  • · Replies 2 ·
Replies
2
Views
1K
Replies
3
Views
2K
  • · Replies 9 ·
Replies
9
Views
3K